For each of the following, draw a diagram or use words to explain your answer.

A. How many 2/3 are in 2?


B. How many 1/10 are in 3?

Answers

Answer 1

Answer:

A. 3.    B. 20

Step-by-step explanation:

2 divided by 2/3 is 2/ 2/3 or  6/2   which is 3.

3 divided by 1/10 is  3/ 1/10 or 30.


Related Questions

please solve the question to get brainliest with explanation​

Answers

Answer:

14) B. 9

15) C. (7 x 3)cm

Step-by-step explanation:

14) The area of of parallelogram ABCD is 63 and the altitude is 7

divide 63 by 7 = 9

15) The length of OC is twice the length of EO so CE = 1(7) + 2(7) = 3(7)

Answer:

14. B

15. C

Step-by-step explanation:

Question 14

Area of a parallelogram = base × altitude

Given:

area ABEF = 63 cm²base = ABaltitude = 7 cm

Substituting the given values into the formula and solve for AB:

⇒ Area = base × altitude

⇒ 63 = AB × 7

[tex]\sf \implies AB=\dfrac{63}{7}[/tex]

⇒ AB = 9cm

---------------------------------------------------------------------------------------------------

Question 15

Median:  the line segment from the vertex to the midpoint of the opposite side.

Median Theorem:  the medians of a triangle intersect at a point called the centroid.  The centroid is two-thirds of the distance from the vertices to the midpoint of the opposite sides.

Therefore,

[tex]\sf CO=\dfrac23CE \implies EO=\dfrac13CE[/tex]

Given:

EO = 7cm

Substituting the given values into the formula and solve for CE:

[tex]\sf \implies EO=\dfrac13CE[/tex]

[tex]\sf \implies 7=\dfrac13CE[/tex]

⇒ CE = (7 x 3) cm

The total amount of carbohydrates in 3.5 bagels is 167.37 grams. How many grams of carbohydrates would one bagel have?

Answers

Answer:

47.82 grams of carbohydrates.

Step-by-step explanation:

3.5 bagels have 167.37 grams of carbohydrates.

1 bagle = ???

In order for 3.5 to be 1 (just as we are asked) we need to divide by 3.5 both the amount of bagles and the amount of carbohydrates.

3.5 / 3.5 = 167.37 / 3.5

1 bagle = 47.82 grams of carbohydrates.

Determine whether each ordered pair is a solution to the inequality x+y<−1.

Answers

Answer:

Choice A.  (10, -1)
Choice B.  (-8, 9)
Choice D.  (6, -3)

Step-by-step explanation:

If we plug the coordinates of point A into the inequality, then we get,
x+y > -2

10 + (-1) > -2
9 > -2
That last inequality is a true statement since 9 is to the right of -2 on the number line. That means (10,-1) is a solution. Choice A is one of the answers.
Choices B and D are also answers
for similar reasons.
Something like choice C is not a solution because
x+y > -2
-1+(-9) > -2
-10 > -2
Which is false.
You should find that choice E is false as well.

If you graphed the inequality and all of the points mentioned (see below), then you can visually confirm the answers. Notice how points A, B and D are in the blue shaded region which is the solution set.
The point E on the boundary does not count as a solution. This is due to the lack of "or equal to" portion of the inequality sign. That visually shows point E is not a solution. Point C isn't a solution either as it's nowhere near the blue shaded region.

Karen drew a triangle with side lengths 3 centimeters, 4 centimeters, and 5 centimeters. What is the perimeter of the triangle?​

Answers

Answer:

12 cm

Explanation:

The formula or method to find the area of any shape is to go around the shape.

In this case, they are 3, 4 and 5.

p = 3 + 4 + 5

p = 12 cm

19 pints converted to gallons

Answers

Answer:

2.375

Step-by-step explanation:

Divide the volume value by 8

and you get the answer 2.375

Hope it helps

TC

have a great time

What numbers round to 200,000 when rounded to the nearest hundred thousand? Select all that apply.

Answers

Answers: Choice C and Choice D

165,400 and 220,574

=================================================

Explanation:

The value in the hundred thousands digit of 165,400 is "1". Just to right of it is "6", which is five or more, meaning we bump that "1" up to "2". Everything else gets replaced with a zero.

The number 165,400 rounds to 200,000 when rounding to the nearest hundred thousand. The number 165,400 is closer to 200,000 than it is to 100,000.

-------------------------

In 220,574 we have the first "2" in the  hundred thousands digit with another "2" just to the right of it. Since that second "2" is not five or more, we replace everything with zeros to get 200,000. We won't bump that first "2" up by one.

220,574 is closer to 200,000 than it is to 300,000.

These two sections help show why choices C and D are the answers.

--------------------------

Something like 142,026 rounds to 100,000 for similar reasoning as the previous section. So we can rule out choice B.

Choice A rounds to 100,000 as well, so we can rule it out also.

Choice D rounds to 1,000,000 which means we cross it off the list.

The sum of three consecutive even integers is 108. What is the largest number?
34
35
37
38

Answers

Answer:

38

Step-by-step explanation:38 because  108. x = 17. 2x + 4 = 34 + 4 = 38

Jorge's math grade is calculated using a weighted average. Quizzes are worth 30%, homework is worth 10%, unit tests are worth 40%, and the final exam is worth 20%.

Jorge earned an average of 92% on his quizzes, 98% on his homework, 91% on his unit tests, and a score of 95% on his final exam.

What is Jorge's final math grade?

Answers

By rewriting the weights in decimal form and applying them to the correspondent percentages, we will see that the final grade is 92.8%.

What is Jore's final math grade?

The final grade will be given by:

G = a₁*x₁ + a₂*x₂ + ...

Where the values "a" are the weights, and the values "x" are the averages of Jeorge.

We know that the weights are:

30% on Quizzes.10% on homework40% on unit test.20% on the final exam.

Then we can write the weights in decimal form:

a₁ = 0.3

a₂ = 0.1

a₃ = 0.4

a₄ = 0.2

And the scores are:

92% on quizzes.98% on homework91% on unit tests95% on final exam.

So the final math grade will be:

G = 0.3*92 + 0.1*98 + 0.4*91 + 0.2*95 = 92.8%

If you want to learn more about percentages, you can read:

https://brainly.com/question/843074

Answer:

the answer is 92.8%

Step-by-step explanation:

Find the diameter of a circle with an
area of 99 yd?. Use your calculator's
value of pi and round your answer to
the nearest tenth

Answers

Answer: 832

Step-by-step explanation:

trust i got correct

I WILL MARK YOU THE BRAINLIEST LINKS WILL BE REPORTED AND DELETED !
What is the value of 7p7 ?
A: 1
B: 14
C: 49
D: 5040

Answers

Answer:

the answer is D:5040

Step-by-step explanation:

The answer to your question would be D. 5040! Hope this helps - Have an amazing rest of your day :)

A group of adults plus one child attend a movie at a movie theater. Tickets cost $9 for adults and $4 for children. The total cost for the movie is $85. Enter an equation to find the number of adults in the group. The equation + a = can help find out how many adults were in the group.

Answers

Answer:

There is 8 adults and 1 child

Step-by-step explanation:

$78 - $6 = $72

$72 divided by 9 = 8

you would add nine 8 times and add 6 to it then it would be $78
Hope this helps you! :)

x – 5 ÷ 3 + b
6b + 15b ÷ 3
c - 2c × 4

PLEASE ANSWER ITS CONFUSING

Answers

Hey there!


Basically for all of your equations/questions you have to COMBINE the LIKE TERMS (if they are any in the particular equation you have)

/\/\/\/\/\/\/\/\/\/\/\/\/\/\/\/\/\/\/\/\/\/\/\/\/\/\/\/\/\

QUESTION/EQUATION #1

x - 5 ÷ 3 + b

We pretty much CANNOT do anything to this equation because it doesn’t have that many like terms.

So, thus your answer will most likely be/

[b + x - 5 ÷ 3]
~~~~~~~~~~~~~~~~~~~~~~~~~~~~~~~~~~~

QUESTION/EQUATION #2

6b + 15b ÷ 3

= 21b ÷ 3

= 7b

Thus, your answer is: [7b]
~~~~~~~~~~~~~~~~~~~~~~~~~~~~~~~~~~~
QUESTION/EQUATION #3

c - 2c × 4

REWRITE

= 1c - 2c × 4

COMBINE the LIKE TERMS

= 1c - 7

= -7c
Thus, your answer is: [-7c]

Good luck on your assignment & enjoy your day!


~Amphitrite1040:)

Steve wants to download a selection of new music. How many ways can Steve select three rock songs, four alternative songs, and three rap songs from a list of eleven rock songs, five alternative songs, and four rap songs?

Answers

Using the combination formula, it is found that there are 3300 ways to choose the songs.

The order in which the musics are chosen is not important, hence the combination formula is used to solve this question.

What is the combination formula?

[tex]C_{n,x}[/tex] is the number of different combinations of x objects from a set of n elements, given by:

[tex]C_{n,x} = \frac{n!}{x!(n-x)!}[/tex]

For the rock songs, we have three from a set of eleven, hence:

[tex]C_{11,3} = \frac{11!}{3!8!} = 165[/tex]

For the alternative songs, we have four from a set of five, hence:

[tex]C_{5,4} = \frac{5!}{4!1!} = 5[/tex]

For the rap songs, we have three from a set of four, hence:

[tex]C_{4,3} = \frac{4!}{3!1!} = 4[/tex]

Since the songs are independent, the total number of ways is given by:

[tex]T = 165 \times 5 \times 4 = 3300[/tex]

More can be learned about the combination formula at https://brainly.com/question/25821700

I really need help... can someone pls help me

Answers

Volume = length x width x height

Volume of large box = 4 x 3 x 5 = 60 cubic feet.


the volume of the smaller boxes is greater than the volume of the larger box so he cannot fit them all.

find the zeros simplify all irrational and complex solutions

f(x)=x^4-11x^2-80

Answers

Answer:

Step-by-step explanation:

f(x)=x^4-11x^2-80=x^4-16x^2+5x^2-80

=x²(x²-16)+5(x²-16)

=(x²-16)(x²+5)

=(x-4)(x+4)[x²-(-5)]

=(x-4)(x+4)[x^2-(5i²)]

=(x-4)(x+4)(x-√5 i)(x+√5 i)

so zeros are 4,-4,√5 i,-√5 i

Sarah took out a 4-year loan for $17,000 to purchase a car. If the interest rate on the loan is 1.3% compounded monthly, how much will she pay in total over the 4 years? Round your answer to the hundredths place.

Answers

Answer:

ummm maybe 960

Step-by-step explanation:

I d k... if he made no payments it'd still be 8000 right?

Solve the equation.

−10x + 1 + 7x = 37

x = −15
x = −12
x = 12
x = 15

Answers

X=-12

-10x+1+7x=37
(-10x+7x)+1=37
-3x=37-1
-3x=36
X=36/-3
X=-12
Check
-10(-12)+1+7(-12)=37
120+1-84=37
37=37

(9 x 7) + 7 - 8 = ?
EXPLAIN HOW!

I give you 10 pts

Answers

Answer:

[tex]\huge\boxed{\bf\:62}[/tex]

Step-by-step explanation:

[tex](9*7) + 7 - 8 = ?[/tex]

We can solve this by using the PEMDAS concept where,

P = Parentheses E = Exponents M = Multiplication D = Division A = Addition S = Subtraction

So, to solve this, we need to solve the numbers in the parentheses by doing the requested operation (here it is, mutiplucation).

[tex](9*7) + 7 - 8\\= (63) + 7 - 8[/tex]

Now, by looking at the PEMDAS rule, we can see that the next step is addition (as, we don't have to divide in this question).

[tex](63) + 7 - 8\\= 63 + 7 - 8\\= 70 - 8[/tex]

Now, subtract 8 from 70 .

[tex]70 - 8\\= \boxed{\bf\:62}[/tex]

[tex]\rule{150pt}{2pt}[/tex]

if josh gives 13 fried fish and molly eats 10 how many left are there

Answers

Answer:

13-10=3 hence, the Q of ans is 3

Josh gives 13 FISH
Molly ate 10 FISH

13 - 10 = 3

So there are 3 left

Question 3 of 5 How many solutions does 6 + = -5 - Lo have? 31 10 ? O A. One solution O B. Infinitely many solutions O C. Two solutions O D. No solutions​

Answers

Answer:

En la Imagen adjunta

Step-by-step explanation:

Toca Para Abrir Imagen

Espero haberte ayudado si es así dame Corona

Answer:

One Solution

Step-by-step explanation:

Given:

6 + x/5 = x/5 - 5 - 3x/10

To Find:

How many solutions?

Solve:

[tex]6+\frac{x}{5}=\frac{x}{5}-5-\frac{3x}{10}[/tex]

[tex]\mathrm{Subtract\:}\frac{x}{5}\mathrm{\:from\:both\:sides}[/tex]

[tex]6+\frac{x}{5}-\frac{x}{5}=\frac{x}{5}-5-\frac{3x}{10}-\frac{x}{5}[/tex]

[tex]\mathrm{Simplify}[/tex]

[tex]6=-5-\frac{3x}{10}[/tex]

[tex]\mathrm{Add\:}5\mathrm{\:to\:both\:sides}[/tex]

[tex]-5-\frac{3x}{10}+5=6+5[/tex]

[tex]\mathrm{Simplify}[/tex]

[tex]-\frac{3x}{10}=11[/tex]

[tex]\mathrm{Multiply\:both\:sides\:by\:}10[/tex]

[tex]10\left(-\frac{3x}{10}\right)=11\cdot \:10[/tex]

[tex]\mathrm{Simplify}[/tex]

[tex]-3x=110[/tex]

[tex]\mathrm{Divide\:both\:sides\:by\:}-3[/tex]

[tex]\frac{-3x}{-3}=\frac{110}{-3}[/tex]

[tex]\mathrm{Simplify}[/tex]

[tex]x=-\frac{110}{3}\;or\;\left(\mathrm{In \;Decimal\;form}:\quad x=-36.66666\dots \right)[/tex]

~lenvy~

the graph of f x shown below resembles the graph of g =x2 but it has been stretched somewhat and shifted which of the following could be the equation of f

Answers

The graph of the function g(x) is a quadratic equation

The possible equation of the function f(x) is f(x) = 2(x - 3)^2 + 3

How to determine the possible equation of f(x)?

From the question, we have:

g(x) = x^2 --- the equation of the graph

The function f(x) is in the first quadrant.

This means that the function g(x) is shifted right and up.

This is represented as:

f(x) = (x - h)^2 + k

When stretched, the function becomes

f(x) = a(x - h)^2 + k

Where:

a > 0

From the list of given options;

The equation that has the form f(x) = a(x - h)^2 + k is

f(x) = 2(x - 3)^2 + 3

Hence, the possible equation of the function f(x) is f(x) = 2(x - 3)^2 + 3

Read more about function transformation at:

https://brainly.com/question/17586310

I need help with this question

Answers

                                                                                                                                                               

Step-by-step explanation:

                           

Question 1) What is the area of triangle BCD to the nearest tenth of a square centimeter? Use special right triangles to help find the height. Show your work. ( Please Do Not Repost Someone Else's Answer That's On Brainly Or Any Other Websites Please If So You'll Be Reported). ​

Answers

Answer:

21.6 cm²

Step-by-step explanation:

Step 1 : Find CB

CB = tan60° x 5CB = √3 x 5CB = 1.73 x 5CB = 8.65 cm

Step 2 : Find Area

Area = 1/2 x Base x HeightArea = 1/2 x 5 x 8.65Area = 2.5 x 8.65Area = 21.625Area = 21.6 cm²

Fill in the missing term on each line. Simplify any fractions.
4t + 11 = 19
4t =
Subtract 11 from both sides
t =
Divide both sides by 4

Answers

Answer:

Step-by-step explanation:

4t + 11 = 19

Subtract 11 from both sides

4t = 19 - 11

4t = 8

Divide both sides by 4

t = 8/4

t = 2

Please Help i Need this asap and i dont know

Answers

Answer: Its 1,110

Step-by-step explanation:

Need Help Please!!

Determining the price for a product can be tricky! Companies often have to juggle several variables in order to find the price that maximizes their revenue. Currently, the price of a product is set at $100. The company is getting about 20 customers per week at this rate. The company has done some research and found that they lose two customers for every $5 they increase their price.

Let the variable ”x” represent the number of times the company increases the price by $5. What is the price of the product when x= 3?

Answers

Check the picture below.

a student gets 11 out of 16 problems correct on a homework assignment. what percent of the assignment did the student get corret?. thanks if someone answers this​

Answers

Answer:

68.75 Percent

Step-by-step explanation:

11/16 =0.6875

0.6875 into percent is 68.75

If the total profit per month is Rs 200,000 and the revenue per month can be obtained twice the cost per month minus 50,000, find the revenue and cost per month

Answers

Step-by-step explanation:

200,000 X 50,000 = 100000000

If the total profit per month is Rs 200,000 and the revenue per month can be obtained twice the cost per month minus 50,000 then  the revenue per month is 2083 and the cost per month is 25000

What is Equation?

Two or more expressions with an Equal sign is called as Equation.

Given,

If the total profit per month is Rs 200,000

Revenue per month can be obtained twice the cost per month minus 50,000

2x-50000

2x=50000

x=25000

So cost per month is 25000

The revenue per month is 25000/12

2083

Hence, the revenue per month is 2083 and the cost per month is 25000

To learn more on Equation:

https://brainly.com/question/10413253

#SPJ2

does anybody know how to do this please help meee

Answers

Answer:

A

Step-by-step explanation: I believe it's A because It's the opposite but I'm not 100% certain, because I don't remember inverse statements.

If measure of angle T= (2x + 85), measure of angle V= (5x + 29), and
the sum of the measures of the angles is
93°, find the measure of each angle.

Answers

Answers:

Angle T = 79 degrees

Angle V = 14 degrees

=========================================================

Work Shown:

T + V = 93

(2x+85) + (5x+29) = 93

7x+114 = 93

7x = 93-114

7x = -21

x = -21/7

x = -3

This x value is used to find each angle mentioned.

Angle T = 2x+85 = 2(-3)+85 = -6+85 = 79 degreesAngle V = 5x+29 = 5(-3)+29 = -15+29 = 14 degrees

As a check, T+V = 79+14 = 93 which confirms our answers.

Answer:

An alternate interior angle, c, 180 degrees

Step-by-step explanation:

did it

Other Questions
Ken wants to build a table and put a border around it. The table and border must have an area of 3,276 square inches. The table is 36 inches wide and 72 inches long without the border. Which quadratic equation can be used to determine the thickness of the border, x? 4x2 216x 2,592 = 0 4x2 216x 684 = 0 2x2 216x 3,276 = 0 x2 108x 3,276 = 0. According to freud, the conflicts that arise during the oedipus conflict are pushed out of awareness by the use of a defense mechanism called What kinds of conflicts resulted from the global confrontation. To conserve energy, the fuel administration shortened workweeks for factories that did not make war materials and introduced. Define nondisjunction. Is this a beneficial process? Explain. N2 (g) + 3H2 (g) 2NH3 (g) + Energy Complete the table below by filling in "right" or "left" for the equilibrium shift and "increases" or "decreases" for the concentration of reactants and products. How do you find the mechanical advantage of a gear? A cube has an edge length of 12 meters. What is its volume, in cubic meters? 15.4 segment relationships in circles homework answers The Length Of A Room is two time its breadth and breadth is two time its height If The volume of room is 512m2 what is the cost of plastering its wall at rs 5.50 per m2 What is the value of x? sin(x 37)=cos(2x 8) enter your answer in the box. x = Please help me on this Subtract 17ab from 2ab A window in the shape of a parallelogram had a base of 46 inches and a height of 45 inches. What is the area of the window? The spread of Islam in West Africa before 1800 was mainly through......... The program can both _________ and entertain young audiences. In "a worn path," phoenix jackson ultimately shows herself to be someone whose actions are all done out of __________. question 1 options: selfishness and bitterness dedication and love confusion and anger confidence and faith The highest peak in North America, Mt. McKinley (or Denali), is located in the state of __________. A. Nevada B. Washington C. Colorado D. Alaska Please select the best answer from the choices provided. A B C D ASAP HELPThe troposphere revives most of its heat energy from the blank Can someone please help me find the ordered pairs for U and T?thanks!